Xem bài viết đơn
Old 12-07-2012, 09:55 PM   #8
minhphuc.v
+Thành Viên+
 
Tham gia ngày: Jul 2012
Bài gởi: 5
Thanks: 1
Thanked 2 Times in 2 Posts
Dùng BĐT Holder ta có
$$(1+1)^1(1+a_2)^2(1+a_3)^3...(1+a_n)^n \ge (1+a_2.a_3...a_n)^{\frac{n(n+1)}{2}}$$
$$=2^{\frac{n(n+1)}{2}} = \left( 2^{\frac{(n+1)}{2}}\right)^n \ge ( 2\ln2.n )^n=n^n(2\ln2)^n > 2n^n, (n \ge 3)$$
Vì ta có hàm số $$f(x)=2^{\frac{x+1}{2}}-2\ln2.x \ge 0, \forall x \ge 3$$
[RIGHT][I][B]Nguồn: MathScope.ORG[/B][/I][/RIGHT]
 

thay đổi nội dung bởi: minhphuc.v, 12-07-2012 lúc 10:05 PM
minhphuc.v is offline   Trả Lời Với Trích Dẫn
The Following User Says Thank You to minhphuc.v For This Useful Post:
9A1 (13-07-2012)
 
[page compression: 7.85 k/8.97 k (12.48%)]